Exam 2

Réussis tes devoirs et examens dès maintenant avec Quizwiz!

A client is receiving interleukins along with chemotherapy. What assessment by the nurse takes priority? a. Blood pressure b. Lung assessment c. Oral mucous membranesd. Skin integrity

A Interleukins can cause capillary leak syndrome and fluid shifting, leading to intravascular volume depletion. Although all assessments are important in caring for clients with cancer, blood pressure and other assessments of fluid status take priority.

The nurse caring for oncology clients knows that which form of metastasis is the most common? a. Bloodborne b. Direct invasion c. Lymphatic spread d. Via bone marrow

ANS: A Bloodborne metastasis is the most common way for cancer to metastasize. Direct invasion and lymphatic spread are other methods. Bone marrow is not a medium in which cancer spreads, although cancer can occur in the bone marrow.

A child was abducted and raped. Which personal reaction by the nurse could interfere with the childs care? a. Anger b. Concern c. Empathy d. Compassion

ANS: A Feelings of empathy, concern, and compassion are helpful. Anger, on the other hand, may make objectivity impossible.

A depressed patient says, Nothing matters anymore. What is the most appropriate response by the nurse? a. Are you having thoughts of suicide? b. I am not sure I understand what you are trying to say. c. Try to stay hopeful. Things have a way of working out. d. Tell me more about what interested you before you began feeling depressed.

ANS: A The nurse must make overt what is covert; that is, the possibility of suicide must be openly addressed. Often, patients feel relieved to be able to talk about suicidal ideation.

A woman has been using acupuncture to treat the nausea and vomiting caused by the side effects of chemotherapy for breast cancer. Which conditions would cause the nurse to recommend against further use of acupuncture? (Select all that apply.) a. Lymphedema b. Bleeding tendencies c. Low white blood cell count d. Elevated serum calcium e. High platelet count

ANS: A, B, C Acupuncture could be unsafe for the client if there is poor drainage of the extremity with lymphedema or if there was a bleeding tendency and low white blood cell count. Coagulation would be compromised with a bleeding disorder, and the risk of infection would be high with the use of needles. An elevated serum calcium and high platelet count would not have any contraindication for acupuncture.

A person was abducted and raped at gunpoint by an unknown assailant. Which interventions should the nurse use while caring for this person in the emergency department? Select all that apply. a. Allow the person to talk at a comfortable pace. b. Pose questions in nonjudgmental, empathic ways. c. Place the person in a private room with a caregiver. d. Reassure the person that a family member will arrive as soon as possible. e. Invite family members to the examination room and involve them in taking the history. f. Put an arm around the person to offer reassurance that the nurse is caring and compassionate.

ANS: A, B, C Neutral, nonjudgmental care and emotional support are critical to crisis management for the victim of rape. The rape victim should have privacy but not be left alone. Some rape victims prefer not to have family members involved. The patients privacy may be compromised by the presence of family. The rape victims anxiety may escalate when he or she is touched by a stranger, even when the stranger is a nurse.

After a breast examination, the nurse is documenting assessment findings that indicate possible breast cancer. Which abnormal findings need to be included as part of the clients electronic medical record? (Select all that apply.) a. Peau dorange b. Dense breast tissue c. Nipple retraction d. Mobile mass at two oclock e. Nontender axillary nodes

ANS: A, C, D In the documentation of a breast mass, skin changes such as dimpling (peau dorange), nipple retraction, and whether the mass is fixed or movable are charted. The location of the mass should be stated by the face of a clock. Dense breast tissue and nontender axillary nodes are not abnormal assessment findings that may indicate breast cancer.

Because an intervention is required to control a patients aggressive behavior, a critical incident debriefing takes place. Which topics should be the focus of the discussion? Select all that apply. a. Patient behavior associated with the incident b. Genetic factors associated with aggression c. Intervention techniques used by staff d. Effect of environmental factors e. Review of theories of aggression

ANS: A, C, D The patients behavior, the intervention techniques used, and the environment in which the incident occurred are important to establish realistic outcomes and effective nursing interventions. Discussing the views about the theoretical origins of aggression is less effective.

When an emergency department nurse teaches a victim of the rape about reactions that may occur during the long-term reorganization phase, which symptoms should be included? Select all that apply. a. Development of fears and phobias b. Decreased motor activity c. Feelings of numbness d. Flashbacks, dreams e. Syncopal episodes

ANS: A, C, D These reactions are common to the long-term reorganization phase. Victims of rape frequently have a period of increased motor activity rather than decreased motor activity during the long-term reorganization phase. Syncopal episodes are not expected.

Which changes in brain biochemical function is most associated with suicidal behavior? a. Dopamine excess b. Serotonin deficiency c. Acetylcholine excess d. Gamma-aminobutyric acid deficiency

ANS: B Research suggests that low levels of serotonin may play a role in the decision to commit suicide. The other neurotransmitter alterations have not been implicated in suicidal crises.

A client is having a catheter placed in the femoral artery to deliver yttrium-90 beads into a liver tumor. What action by the nurse is most important? a. Assessing the clients abdomen beforehand b. Ensuring that informed consent is on the chart c. Marking the clients bilateral pedal pulses d. Reviewing client teaching done previously

ANS: B This is an invasive procedure requiring informed consent. The nurse should ensure that consent is on the chart. The other actions are also appropriate but not the priority.

A client is receiving rituximab (Rituxan) and asks how it works. What response by the nurse is best? a. It causes rapid lysis of the cancer cell membranes. b. It destroys the enzymes needed to create cancer cells. c. It prevents the start of cell division in the cancer cells. d. It sensitizes certain cancer cells to chemotherapy.

ANS: C Rituxan prevents the initiation of cancer cell division. The other statements are not accurate.

A person intentionally overdoses on antidepressant drugs. Which nursing diagnosis has the highest priority? a. Powerlessness b. Social isolation c. Risk for suicide d. Ineffective management of the therapeutic regimen

ANS: C This diagnosis is the only one with life-or-death ramifications and is therefore higher in priority than the other options.

A nurse cares for a rape victim who was given flunitrazepam (Rohypnol) by the assailant. Which intervention has priority? Monitoring for: a. coma. b. seizures. c. hypotonia. d. respiratory depression.

ANS: D Monitoring for respiratory depression takes priority over hypotonia, seizures, or coma in this situation.

A person attempts suicide by overdose, is treated in the emergency department, and then hospitalized. What is the best initial outcome? The patient will: a. verbalize a will to live by the end of the second hospital day. b. describe two new coping mechanisms by the end of the third hospital day. c. accurately delineate personal strengths by the end of first week of hospitalization. d. exercise suicide self-restraint by refraining from gestures or attempts to kill self for 24 hours.

ANS: D Suicide self-restraint relates most directly to the priority problem of risk for self-directed violence. The other outcomes are related to hope, coping, and self-esteem.

An adult has recently been absent from work for 3-day periods on several occasions. Each time, the individual returns to work wearing dark glasses. Facial and body bruises are apparent. What is the occupational health nurses priority assessment? a. Interpersonal relationships b. Work responsibilities c. Socialization skills d. Physical injuries

ANS: D The individual should be assessed for possible battering. Physical injuries are abuse indicators and are the primary focus for assessment. No data support the other options.

Information from a patients record that indicates marginal coping skills and the need for careful assessment of the risk for violence is a history of: a. childhood trauma. b. family involvement. c. academic problems. d. substance abuse.

ANS: D The nurse should suspect marginal coping skills in a patient with substance abuse. He or she is often anxious, may be concerned about inadequate pain relief, and may have a personality style that externalizes blame. The incorrect options do not signal as high a degree of risk as chemical dependence

Which statement about carcinogenesis is accurate? a. An initiated cell will always become clinical cancer. b. Cancer becomes a health problem once it is 1 cm in size. c. Normal hormones and proteins do not promote cancer growth. d. Tumor cells need to develop their own blood supply.

ANS: D Tumors need to develop their own blood supply through a process called angiogenesis. An initiated cell needs a promoter to continue its malignant path. Normal hormones and proteins in the body can act as promoters. A 1-cm tumor is a detectable size, but other events have to occur for it to become a health problem.

A client is receiving chemotherapy through a peripheral IV line. What action by the nurse is most important? a. Assessing the IV site every hour b. Educating the client on side effects c. Monitoring the client for nausea d. Providing warm packs for comfort

A Intravenous chemotherapy can cause local tissue destruction if it extravasates into the surrounding tissues. Peripheral IV lines are more prone to this than centrally placed lines. The most important intervention is prevention, so the nurse should check hourly to ensure the IV site is patent, or frequently depending on facility policy. Education and monitoring for side effects such as nausea are important for all clients receiving chemotherapy. Warm packs may be helpful for comfort, but if the client reports that an IV site is painful, the nurse needs to assess further.

The feeling experienced by a patient that should be assessed by the nurse as most predictive of elevated suicide risk is: a. hopelessness. b. sadness. c. elation. d. anger.

ANS: A Of the feelings listed, hopelessness is most closely associated with increased suicide risk. Depression, aggression, impulsivity, and shame are other feelings noted as risk factors for suicide.

A client with a history of prostate cancer is in the clinic and reports new onset of severe low back pain. What action by the nurse is most important? a. Assess the clients gait and balance. b. Ask the client about the ease of urine flow. c. Document the report completely. d. Inquire about the clients job risks.

ANS: A This client has manifestations of spinal cord compression, which can be seen with prostate cancer. This may affect both gait and balance and urinary function. For client safety, assessing gait and balance is the priority. Documentation should be complete. The client may or may not have occupational risks for low back pain, but with his history of prostate cancer, this should not be where the nurse starts investigating.

. Which central nervous system structures are most associated with anger and aggression? Select all that apply. a. Amygdala b. Cerebellum c. Basal ganglia d. Temporal lobe e. Parietal lobe

ANS: A, D The amygdala mediates anger experiences and helps a person judge an event as either rewarding or aversive. The temporal lobe, which is part of the limbic system, also plays a role in aggressive behavior. The cerebellum manages equilibrium, muscle tone, and movement. The basal ganglia are involved in movement. The parietal lobe is involved in interpreting sensations.

What feelings are most commonly experienced by nurses working with abusive families? a. Outrage toward the victim and sympathy for the abuser b. Sympathy for the victim and anger toward the abuser c. Unconcern for the victim and dislike for the abuser d. Vulnerability for self and empathy with the abuser

ANS: B Intense protective feelings, sympathy for the victim, and anger and outrage toward the abuser are common emotions of a nurse working with an abusive family.

Which activities are in the scope of practice of a sexual assault nurse examiner? Select all that apply. a. Requiring HIV testing of a victim b. Collecting and preserving evidence c. Providing long-term counseling for rape victims d. Obtaining signed consents for photographs and examinations e. Providing pregnancy and sexually transmitted disease prophylaxis

ANS: B, D, E HIV testing is not mandatory for a victim of sexual assault. Long-term counseling would be provided by other members of the team. The other activities would be included within this practice role.

An adult has recently been absent from work for 3-day periods on several occasions. Each time, this person returns to work wearing dark glasses. Facial and body bruises are apparent. What is the occupational health nurses priority question? a. Do you drink excessively? b. Did your partner beat you? c. How did this happen to you? d. What did you do to deserve this?

ANS: C Obtaining the persons explanation is necessary. If the explanation does not match the injuries or if the victim minimizes the injuries, abuse should be suspected.

A nurse is assessing a client with glioblastoma. What assessment is most important? a. Abdominal palpation b. Abdominal percussion c. Lung auscultation d. Neurologic examination

ANS: D A glioblastoma arises in the brain. The most important assessment for this client is the neurologic examination.

The nursing diagnosis rape trauma syndrome applies to a rape victim in the emergency department. Which outcome should occur before the patients discharge? a. Patient states, I feel safe and entirely relaxed. b. Memory of the rape is less vivid and frightening. c. Physical symptoms of pain and discomfort are no longer present. d. Patient agrees to keep a follow-up appointment with the rape crisis center.

ANS: D Agreeing to keep a follow-up appointment is a realistic short-term outcome. The incorrect options are unlikely to occur during the limited time the victim is in the emergency department.

A student nurse asks the nursing instructor what apoptosis means. What response by the instructor is best? a. Growth by cells enlarging b. Having the normal number of chromosomes c. Inhibition of cell growth d. Programmed cell death

ANS: D Apoptosis is programmed cell death. With this characteristic, organs and tissues function with cells that are at their peak of performance. Growth by cells enlarging is hyperplasia. Having the normal number of chromosomes is euploidy. Inhibition of cell growth is contact inhibition.

Which behavior best demonstrates aggression? a. Stomping away from the nurses station, going to another room, and grabbing a snack from another patient. b. Bursting into tears, leaving the community meeting, and sitting on a bed hugging a pillow and sobbing. c. Telling the primary nurse, I felt angry when you said I could not have a second helping at lunch. d. Telling the medication nurse, I am not going to take that or any other medication you try to give me.

ANS: A Aggression is harsh physical or verbal action that reflects rage, hostility, and the potential for physical or verbal destructiveness. Aggressive behavior violates the rights of others. The incorrect options do not feature violation of anothers rights.

A nurse working with clients who experience alopecia knows that which is the best method of helping clients manage the psychosocial impact of this problem? a. Assisting the client to pre-plan for this event b. Reassuring the client that alopecia is temporary c. Teaching the client ways to protect the scalp d. Telling the client that there are worse side effects

ANS: A Alopecia does not occur for all clients who have cancer, but when it does, it can be devastating. The best action by the nurse is to teach the client about the possibility and to give the client multiple choices for preparing for this event. Not all clients will have the same reaction, but some possible actions the client can take are buying a wig ahead of time, buying attractive hats and scarves, and having a hairdresser modify a wig to look like the clients own hair. Teaching about scalp protection is important but does not address the psychosocial impact. Reassuring the client that hair loss is temporary and telling him or her that there are worse side effects are both patronizing and do not give the client tools to manage this condition.

A patient tells the nurse, My husband is abusive most often when he drinks too much. His family was like that when he was growing up. He always apologizes and regrets hurting me. What risk factor was most predictive for the husband to become abusive? a. History of family violence b. Loss of employment c. Abuse of alcohol d. Poverty

ANS: A An abuse-prone individual is an individual who has experienced family violence and was often abused as a child. This phenomenon is part of the cycle of violence. The other options may be present but are not as predictive.

Which is an effective nursing intervention to assist an angry patient to learn to manage anger without violence? a. Help the patient identify a thought that increases anger, find proof for or against the belief, and substitute reality-based thinking. b. Provide negative reinforcement such as restraint or seclusion in response to angry outbursts, whether or not violence is present. c. Use aversive conditioning, such as popping a rubber band on the wrist, to help extinguish angry feelings. d. Administer an antipsychotic or antianxiety medication when the patient feels angry.

ANS: A Anger has a strong cognitive component; therefore, using cognition to manage anger is logical. The incorrect options do nothing to help the patient learn anger management.

Which statement provides the best rationale for why a nurse should closely monitor a severely depressed patient during antidepressant medication therapy? a. As depression lifts, physical energy becomes available to carry out suicide. b. Suicide may be precipitated by a variety of internal and external events. c. Suicidal patients have difficulty using social supports. d. Suicide is an impulsive act.

ANS: A Antidepressant medication has the objective of relieving depression. The risk for suicide is greater as the depression lifts, primarily because the patient has more physical energy at a time when he or she may still have suicidal ideation. The other options have little to do with nursing interventions relating to antidepressant medication therapy.

The nurse working with oncology clients understands that which age-related change increases the older clients susceptibility to infection during chemotherapy? a. Decreased immune function b. Diminished nutritional stores c. Existing cognitive deficits d. Poor physical reserves

ANS: A As people age, there is an age-related decrease in immune function, causing the older adult to be more susceptible to infection than other clients. Not all older adults have diminished nutritional stores, cognitive dysfunction, or poor physical reserves.

An unconscious person is brought to the emergency department by a friend. The friend found the person in a bedroom at a college fraternity party. Semen is observed on the persons underclothes. The priority actions of staff members should focus on: a. maintaining the airway. b. preserving rape evidence. c. obtaining a description of the rape. d. determining what drug was ingested.

ANS: A Because the patient is unconscious, the risk for airway obstruction is present. The incorrect options are of lower priority than preserving physiologic functioning.

A group of nursing students has entered a futuristic science contest in which they have developed a cure for cancer. Which treatment would most likely be the winning entry? a. Artificial fibronectin infusion to maintain tight adhesion of cells b. Chromosome repair kit to halt rapid division of cancer cells c. Synthetic enzyme transfusion to allow rapid cellular migration d. Telomerase therapy to maintain chromosomal immortality

ANS: A Cancer cells do not have sufficient fibronectin and so do not maintain tight adhesion with other cells. This is part of the mechanism of metastasis. Chromosome alterations in cancer cells (aneuploidy) consist of having too many, too few, or altered chromosome pairs. This does not necessarily lead to rapid cellular division. Rapid cellular migration is part of metastasis. Immortality is a characteristic of cancer cells due to too much telomerase.

A nurse in the oncology clinic is providing preoperative education to a client just diagnosed with cancer. The client has been scheduled for surgery in 3 days. What action by the nurse is best? a. Call the client at home the next day to review teaching. b. Give the client information about a cancer support group. c. Provide all the preoperative instructions in writing. d. Reassure the client that surgery will be over soon.

ANS: A Clients are often overwhelmed at a sudden diagnosis of cancer and may be more overwhelmed at the idea of a major operation so soon. This stress significantly impacts the clients ability to understand, retain, and recall information. The nurse should call the client at home the next day to review the teaching and to answer questions. The client may or may not be ready to investigate a support group, but this does not help with teaching. Giving information in writing is important (if the client can read it), but in itself will not be enough. Telling the client that surgery will be over soon is giving false reassurance and does nothing for teaching.

A client asks the nurse if eating only preservative- and dye-free foods will decrease cancer risk. What response by the nurse is best? a. Maybe; preservatives, dyes, and preparation methods may be risk factors. b. No; research studies have never shown those things to cause cancer. c. There are other things you can do that will more effectively lower your risk. d. Yes; preservatives and dyes are well known to be carcinogens.

ANS: A Dietary factors related to cancer development are poorly understood, although dietary practices are suspected to alter cancer risk. Suspected dietary risk factors include low fiber intake and a high intake of red meat or animal fat. Preservatives, preparation methods, and additives (dyes, flavorings, sweeteners) may have cancer-promoting effects. It is correct to say that other things can lower risk more effectively, but this does not give the client concrete information about how to do so, and also does not answer the clients question.

A client is placed on a medical regimen of doxorubicin (Adriamycin), cyclophosphamide (Cytoxan), and fluorouracil (5-FU) for breast cancer. Which side effect seen in the client should the nurse report to the provider immediately? a. Shortness of breath b. Nausea and vomiting c. Hair loss d. Mucositis

ANS: A Doxorubicin (Adriamycin) can cause cardiac problems with symptoms of extreme fatigue, shortness of breath, chronic cough, and edema. These need to be reported as soon as possible to the provider. Nausea, vomiting, hair loss, and mucositis are common problems associated with chemotherapy regimens.

A nurse works on an oncology unit and delegates personal hygiene to an unlicensed assistive personnel (UAP). What action by the UAP requires intervention from the nurse? a. Allowing a very tired client to skip oral hygiene and sleep b. Assisting clients with washing the perianal area every 12 hours c. Helping the client use a soft-bristled toothbrush for oral care d. Reminding the client to rinse the mouth with water or saline

ANS: A Even though clients may be tired, they still need to participate in hygiene to help prevent infection. The other options are all appropriate.

The staff development coordinator plans to teach use of physical management techniques when patients become assaultive. Which topic should be emphasized? a. Practice and teamwork b. Spontaneity and surprise c. Caution and superior size d. Diversion and physical outlets

ANS: A Intervention techniques are learned behaviors that must be practiced to be used in a smooth, organized fashion. Every member of the intervention team should be assigned a specific task to carry out before beginning the intervention. The other options are useless if the staff does not know how to use physical techniques and how to apply them in an organized fashion.

A patient was responding to auditory hallucinations earlier in the morning. The patient approaches the nurse, shaking a fist and shouting, Back off! and then goes into the day room. As the nurse follows the patient into the day room, the nurse should: a. make sure adequate physical space exists between the nurse and the patient. b. move into a position that allows the patient to be close to the door. c. maintain one arms length distance from the patient. d. sit down in a chair near the patient.

ANS: A Making sure space is present between the nurse and the patient avoids invading the patients personal space. Personal space needs increase when a patient feels anxious and threatened. Allowing the patient to block the nurses exit from the room is not wise. Closeness may be threatening to the patient and provoke aggression. Sitting is inadvisable until further assessment suggests the patients aggression is abating. One arms length is inadequate space.

A person was abducted and raped at gunpoint by an unknown assailant. Which assessment finding best indicates the person is in the acute phase of rape trauma syndrome? a. Confusion and disbelief b. Decreased motor activity c. Flashbacks and dreams d. Fears and phobias

ANS: A Shock, emotional numbness, confusion, disbelief, restlessness, and agitated motor activity depict the acute phase of rape trauma syndrome. Flashbacks, dreams, fears, and phobias occur in the long-term reorganization phase of rape trauma syndrome. Decreased motor activity, by itself, is not indicative of any particular phase.

A woman diagnosed with breast cancer had these laboratory tests performed at an office visit:Alkaline phosphatase 125 U/LTotal calcium 12 mg/dLHematocrit 39%Hemoglobin 14 g/dLWhich test results indicate to the nurse that some further diagnostics are needed? a. Elevated alkaline phosphatase and calcium suggests bone involvement. b. Only alkaline phosphatase is decreased, suggesting liver metastasis. c. Hematocrit and hemoglobin are decreased, indicating anemia. d. The elevated hematocrit and hemoglobin indicate dehydration.

ANS: A The alkaline phosphatase (normal value 30 to 120 U/L) and total calcium (normal value 9 to 10.5 mg/dL) levels are both elevated, suggesting bone metastasis. Both the hematocrit and hemoglobin are within normal limits for females.

A nurse reads on a hospitalized clients chart that the client is receiving teletherapy. What action by the nurse is best? a. Coordinate continuation of the therapy. b. Place the client on radiation precautions. c. No action by the nurse is needed at this time. d. Restrict visitors to only adults over age 18.

ANS: A The client needs to continue with radiation therapy, and the nurse can coordinate this with the appropriate department. The client is not radioactive, so radiation precautions and limiting visitors are not necessary.

A client hospitalized for chemotherapy has a hemoglobin of 6.1 mg/dL. What medication should the nurse prepare to administer? a. Epoetin alfa (Epogen) b. Filgrastim (Neupogen) c. Mesna (Mesnex) d. Oprelvekin (Neumega)

ANS: A The clients hemoglobin is low, so the nurse should prepare to administer epoetin alfa, a colony-stimulating factor that increases production of red blood cells. Filgrastim is for neutropenia. Mesna is used to decrease bladder toxicity from some chemotherapeutic agents. Oprelvekin is used to increase platelet count.

A patient at the emergency department is diagnosed with a concussion. The patient is accompanied by a spouse who insists on staying in the room and answering all questions. The patient avoids eye contact and has a sad affect and slumped shoulders. Assessment of which additional problem has priority? a. Risk of intimate partner violence b. Phobia of crowded places c. Migraine headaches d. Major depression

ANS: A The diagnosis of a concussion suggests violence as a cause. The patient is exhibiting indicators of abuse including fearfulness, depressed affect, poor eye contact, and a possessive spouse. The patient may be also experiencing depression, anxiety, and migraine headaches, but the nurses advocacy role necessitates an assessment for intimate partner violence.

A college student who attempted suicide by overdose is hospitalized. When the parents are contacted, they respond, There must be a mistake. This could not have happened. Weve given our child everything. The parents reaction reflects: a. denial. b. anger. c. anxiety. d. rescue feelings.

ANS: A The parents statements indicate denial. Denial or minimization of suicidal ideation or attempts is a defense against uncomfortable feelings. Family members are often unable to acknowledge suicidal ideation in someone close to them. The feelings suggested in the distractors are not clearly described in the scenario.

An adult attempts suicide after declaring bankruptcy. The patient is hospitalized and takes an antidepressant medication for five days. The patient is now more talkative and shows increased energy. Select the highest priority nursing intervention. a. Supervise the patient 24 hours a day. b. Begin discharge planning for the patient. c. Refer the patient to art and music therapists. d. Consider the discontinuation of suicide precautions.

ANS: A The patient now has more energy and may have decided on suicide, especially considering the history of the prior suicide attempt. The patient is still a suicide risk; therefore, continuous supervision is indicated.

A victim of a violent rape has been in the emergency department for 3 hours. Evidence collection is complete. As discharge counseling begins, the victim says softly, I will never be the same again. I cant face my friends. There is no sense of trying to go on. Select the nurses most important response. a. Are you thinking of suicide? b. It will take time, but you will feel the same as before. c. Your friends will understand when you tell them. d. You will be able to find meaning in this experience as time goes on.

ANS: A The victims words suggest hopelessness. Whenever hopelessness is present, so is the risk for suicide. The nurse should directly address the possibility of suicidal ideation with the victim. The other options attempt to offer reassurance before making an assessment.

Four individuals have given information about their suicide plans. Which plan evidences the highest suicide risk? a. Jumping from a 100-foot-high railroad bridge located in a deserted area late at night b. Turning on the oven and letting gas escape into the apartment during the night c. Cutting the wrists in the bathroom while the spouse reads in the next room d. Overdosing on aspirin with codeine while the spouse is out with friends

ANS: A This is a highly lethal method with little opportunity for rescue. The other options are lower lethality methods with higher rescue potential.

The parents of identical twins ask a nurse for advice. One twin committed suicide a month ago. Now the parents are concerned that the other twin may also have suicidal tendencies. Which comment by the nurse is accurate? a. Genetics are associated with suicide risk. Monitoring and support are important. b. Apathy underlies suicide. Instilling motivation is the key to health maintenance. c. Your child is unlikely to act out suicide when identifying with a suicide victim. d. Fraternal twins are at higher risk for suicide than identical twins.

ANS: A Twin studies suggest the presence of genetic factors in suicide; however, separating genetic predisposition to suicide from predisposition to depression or alcoholism is difficult. Primary interventions can be helpful in promoting and maintaining health and possibly counteracting the genetic load. The incorrect options are untrue statements or oversimplifications.

A new nurse says to a peer, My newest patient is diagnosed with schizophrenia. At least I wont have to worry about suicide risk. Which response by the peer would be most helpful? a. Lets reconsider your plan. Suicide risk is high in patients diagnosed with schizophrenia. b. Suicide is a risk for any patient diagnosed with schizophrenia who uses alcohol or drugs. c. Patients diagnosed with schizophrenia are usually too disorganized to attempt suicide. d. Visual hallucinations often prompt suicide among patients diagnosed with schizophrenia.

ANS: A Up to 10% of patients diagnosed with schizophrenia die from suicide, usually related to depressive symptoms occurring in the early years of the illness. Depressive symptoms are related to suicide among patients diagnosed with schizophrenia. Patients diagnosed with schizophrenia usually have auditory, not visual, hallucinations. Although the use of drugs and alcohol compounds the risk for suicide, it is independent of schizophrenia.

The nurse cares for a victim of a violent sexual assault. What is the most therapeutic intervention? a. Use accepting, nurturing, and empathetic communication techniques. b. Educate the victim about strategies to avoid attacks in the future. c. Discourage the expression of feelings until the victim stabilizes. d. Maintain a matter-of-fact manner and objectivity.

ANS: A Victims require the nurse to provide unconditional acceptance of them as individuals, because they often feel guilty and engage in self-blame. The nurse must be nurturing if the victims needs are to be met and must be empathetic to convey understanding and to promote an establishment of trust.

A patient with suicidal impulses is on the highest level of suicide precautions. Which measures should the nurse incorporate into the patients plan of care? Select all that apply. a. Allow no glass or metal on meal trays. b. Remove all potentially harmful objects from the patients possession. c. Maintain arms length, one-on-one nursing observation around the clock. d. Check the patients whereabouts every hour. Make verbal contact at least three times each shift. e. Check the patients whereabouts every 15 minutes, and make frequent verbal contacts. f. Keep the patient within visual range while he or she is awake. Check every 15 to 30 minutes while the patient is sleeping.

ANS: A, B, C One-on-one observation is necessary for anyone who has limited control over suicidal impulses. Plastic dishes on trays and the removal of potentially harmful objects from the patients possession are measures included in any level of suicide precautions. The remaining options are used in less stringent levels of suicide precautions.

A nurse is preparing to administer IV chemotherapy. What supplies does this nurse need? (Select all that apply.) a. Chemo gloves b. Facemask c. Isolation gown d. N95 respirator e. Shoe covers

ANS: A, B, C The Occupational Safety and Health Administration (OSHA) and the Oncology Nurses Society have developed safety guidelines for those preparing or administering IV chemotherapy. These include double gloves (or chemo gloves), a facemask, and a gown. An N95 respirator and shoe covers are not required.

The nurse working with oncology clients understands that interacting factors affect cancer development. Which factors does this include? (Select all that apply.) a. Exposure to carcinogens b. Genetic predisposition c. Immune function d. Normal doubling time e. State of euploidy

ANS: A, B, C The three interacting factors needed for cancer development are exposure to carcinogens, genetic predisposition, and immune function.

A nurse is providing community education on the seven warning signs of cancer. Which signs are included? (Select all that apply.) a. A sore that does not heal b. Changes in menstrual patterns c. Indigestion or trouble swallowing d. Near-daily abdominal pain e. Obvious change in a mole

ANS: A, B, C, E The seven warning signs for cancer can be remembered with the acronym CAUTION: changes in bowel or bladder habits, a sore that does not heal, unusual bleeding or discharge, thickening or lump in the breast or elsewhere, indigestion or difficulty swallowing, obvious change in a wart or mole, and nagging cough or hoarseness. Abdominal pain is not a warning sign.

A clients family members are concerned that telling the client about a new finding of cancer will cause extreme emotional distress. They approach the nurse and ask if this can be kept from the client. What actions by the nurse are most appropriate? (Select all that apply.) a. Ask the family to describe their concerns more fully. b. Consult with a social worker, chaplain, or ethics committee. c. Explain the clients right to know and ask for their assistance. d. Have the unit manager take over the care of this client and family. e. Tell the family that this secret will not be kept from the client.

ANS: A, B, CT he clients right of autonomy means that the client must be fully informed as to his or her diagnosis and treatment options. The nurse cannot ethically keep this information from the client. The nurse can ask the family to explain their concerns more fully so everyone understands the concerns. A social worker, chaplain, or ethics committee can become involved to assist the nurse, client, and family. The nurse should explain the clients right to know and ask the family how best to proceed. The nurse should not abdicate responsibility for this difficult situation by transferring care to another nurse. Simply telling the family that he or she will not keep this secret sets up an adversarial relationship. Explaining this fact along with the concept of autonomy would be acceptable, but this by itself is not.

A client has thrombocytopenia. What actions does the nurse delegate to the unlicensed assistive personnel (UAP)? (Select all that apply.) a. Apply the clients shoes before getting the client out of bed. b. Assist the client with ambulation. c. Shave the client with a safety razor only. d. Use a lift sheet to move the client up in bed. e. Use the Waterpik on a low setting for oral care.

ANS: A, B, D Clients with thrombocytopenia are at risk of significant bleeding even with minor injuries. The nurse instructs the UAP to put the clients shoes on before getting the client out of bed, assist with ambulation, shave the client with an electric razor, use a lift sheet when needed to reposition the client, and use a soft-bristled toothbrush for oral care.

A nurse directs the intervention team who must take an aggressive patient to seclusion. Other patients were removed from the area. Before approaching the patient, the nurse should ensure that staff take which actions? Select all that apply. a. Remove jewelry, glasses, and harmful items from the patient and staff members. b. Appoint a person to clear a path and open, close, or lock doors. c. Quickly approach the patient, and grab the closest extremity. d. Select the person who will communicate with the patient. e. Move behind the patient to use the element of surprise.

ANS: A, B, D Injury to staff members and to the patient should be prevented. Only one person should explain what will happen and direct the patient; this person might be the nurse or staff member who has a good relationship with the patient. A clear pathway is essential; those restraining a limb cannot use keys, move furniture, or open doors. The nurse is usually responsible for administering the medication once the patient is restrained. Each staff member should have an assigned limb rather than just grabbing the closest limb. This system could leave one or two limbs unrestrained. Approaching in full view of the patient reduces suspicion.

A client has mucositis. What actions by the nurse will improve the clients nutrition? (Select all that apply.) a. Assist with rinsing the mouth with saline frequently. b. Encourage the client to eat room-temperature foods. c. Give the client hot liquids to hold in the mouth. d. Provide local anesthetic medications to swish and spit. e. Remind the client to brush teeth gently after each meal.

ANS: A, B, D, E Mucositis can interfere with nutrition. The nurse can help with rinsing the mouth frequently with water or saline; encouraging the client to eat cool, slightly warm, or room-temperature foods; providing swish-and-spit anesthetics; and reminding the client to keep the mouth clean by brushing gently after each meal. Hot liquids would be painful for the client.

The nurse is taking a history of a 68-year-old woman. What assessment findings would indicate a high risk for the development of breast cancer? (Select all that apply.) a. Age greater than 65 years b. Increased breast density c. Osteoporosis d. Multiparity e. Genetic factors

ANS: A, B, E The high risk factors for breast cancer are age greater than 65 with the risk increasing until age 80; an increase in breast density because of more glandular and connective tissue; and inherited mutations of BRCA1 and/or BRCA2 genes. Osteoporosis and multiparity are not risk factors for breast cancer. A high postmenopausal bone density and nulliparity are moderate and low increased risk factors, respectively.

A nurse assesses five newly hospitalized patients. Which patients have the highest suicide risk? Select all that apply. a. 82-year-old white man b. 17-year-old white female adolescent c. 39-year-old African-American man d. 29-year-old African-American woman e. 22-year-old man with traumatic brain injury

ANS: A, B, E Whites have suicide rates almost twice those of nonwhites, and the rate is particularly high for older adult men, adolescents, and young adults. Other high risk groups include young African-American men, Native-American men, older Asian Americans, and persons with traumatic brain injury.

A client on interferon therapy is reporting severe skin itching and irritation. What actions does the nurse delegate to the unlicensed assistive personnel (UAP)? (Select all that apply.) a. Apply moisturizers to dry skin. b. Apply steroid creams to the skin. c. Bathe the client using mild soap. d. Help the client with a hot water bath. e. Teach the client to avoid sunlight.

ANS: A, C The nurse can delegate applying unscented moisturizer and using mild soap for bathing. Steroid creams are not used for this condition. Hot water will worsen the irritation. Client teaching is a nursing function.

A client receiving chemotherapy has a white blood cell count of 1000/mm3. What actions by the nurse are most appropriate? (Select all that apply.) a. Assess all mucous membranes every 4 to 8 hours. b. Do not allow the client to eat meat or poultry. c. Listen to lung sounds and monitor for cough. d. Monitor the venous access device appearance with vital signs. e. Take and record vital signs every 4 to 8 hours.

ANS: A, C, D, E Depending on facility protocol, the nurse should assess this client for infection every 4 to 8 hours by assessing all mucous membranes, listening to lung sounds, monitoring for cough, monitoring the appearance of the venous access device, and recording vital signs. Eating meat and poultry is allowed.

A nurse assists a victim of intimate partner violence to create a plan for escape if it becomes necessary. The plan should include which components? Select all that apply. a. Keep a cell phone fully charged. b. Hide money with which to buy new clothes. c. Have the telephone number for the nearest shelter. d. Take enough toys to amuse the children for 2 days. e. Secure a supply of current medications for self and children. f. Determine a code word to signal children that it is time to leave. g. Assemble birth certificates, Social Security cards, and licenses.

ANS: A, C, E, F, G The victim must prepare for a quick exit and so should assemble necessary items. Keeping a cell phone fully charged will help with access to support persons or agencies. The individual should be advised to hide a small suitcase containing a change of clothing for self and for each child. Taking a large supply of toys would be cumbersome and might compromise the plan. People are advised to take one favorite small toy or security object for each child, but most shelters have toys to further engage the children. Accumulating enough money to purchase clothing may be difficult.

Which behaviors are most consistent with the clinical picture of a patient who is becoming increasingly aggressive? Select all that apply. a. Pacing b. Crying c. Withdrawn affect d. Rigid posture with clenched jaw e. Staring with narrowed eyes into the eyes of another

ANS: A, D, E Crying and a withdrawn affect are not cited by experts as behaviors indicating that the individual has a high potential to behave violently. The other behaviors are consistent with the increased risk for other-directed violence.

The nurse is formulating a teaching plan according to evidence-based breast cancer screening guidelines for a 50-year-old woman with low risk factors. Which diagnostic methods should be included in the plan? (Select all that apply.) a. Annual mammogram b. Magnetic resonance imaging (MRI) c. Breast ultrasound d. Breast self-awareness e. Clinical breast examination

ANS: A, D, E Guidelines recommend a screening annual mammogram for women ages 40 years and older, breast self-awareness, and a clinical breast examination. An MRI is recommended if there are known high risk factors. A breast ultrasound is used if there are problems discovered with the initial screening or dense breast tissue.

The nursing student learning about cancer development remembers characteristics of normal cells. Which characteristics does this include? (Select all that apply.) a. Differentiated function b. Large nucleus-to-cytoplasm ratio c. Loose adherence d. Nonmigratory e. Specific morphology

ANS: A, D, E Normal cells have the characteristics of differentiated function, nonmigratory, specific morphology, a smaller nucleus-to-cytoplasm ratio, tight adherence, and orderly and well-regulated growth.

A patient with a history of command hallucinations approaches the nurse, yelling obscenities. The patient mumbles and then walks away. The nurse follows. Which nursing actions are most likely to be effective in de-escalating this scenario? Select all that apply. a. State the expectation that the patient will stay in control. b. State that the patient cannot be understood when mumbling. c. Tell the patient, You are behaving inappropriately. d. Offer to provide the patient with medication to help. e. Speak in a firm but calm voice.

ANS: A, D, E Stating the expectation that the patient will maintain control of behavior reinforces positive, healthy behavior and avoids challenging the patient. Offering an as-needed medication provides support for the patient trying to maintain control. A firm but calm voice will likely comfort and calm the patient. Belittling remarks may lead to aggression. Criticism will probably prompt the patient to begin shouting.

A staff nurse tells another nurse, I evaluated a new patient using the SAD PERSONS scale and got a score of 10. Im wondering if I should send the patient home. Select the best reply by the second nurse. a. That action would seem appropriate. b. A score over 8 requires immediate hospitalization. c. I think you should strongly consider hospitalization for this patient. d. Give the patient a follow-up appointment. Hospitalization may be needed soon.

ANS: B A SAD PERSONS scale score of 0 to 5 suggests home care with follow-up. A score of 6 to 8 requires psychiatric consultation. A score over 8 calls for hospitalization.

A client has a platelet count of 9800/mm3. What action by the nurse is most appropriate? a. Assess the client for calf pain, warmth, and redness. b. Instruct the client to call for help to get out of bed. c. Obtain cultures as per the facilitys standing policy. d. Place the client on protective isolation precautions.

ANS: B A client with a platelet count this low is at high risk for serious bleeding episodes. To prevent injury, the client should be instructed to call for help prior to getting out of bed. Calf pain, warmth, and redness might indicate a deep vein thrombosis, not associated with low platelets. Cultures and isolation relate to low white cell counts.

Which assessment finding presents the greatest risk for violent behavior? A patient who: a. is severely agoraphobic b. has a history of intimate partner violence. c. demonstrates bizarre somatic delusions. d. verbalizes hopelessness and powerlessness.

ANS: B A history of prior aggression or violence is the best predictor of patients who may become violent. Patients diagnosed with anxiety disorders are not particularly prone to violence unless panic occurs. Patients experiencing hopelessness and powerlessness may have co-existing anger, but violence is not often demonstrated. Patients experiencing paranoid delusions are at greater risk for violence than those with bizarre somatic delusions.

An intramuscular dose of antipsychotic medication needs to be given to a patient who is becoming increasingly more aggressive. The patient is in the day room. The nurse should enter the day room: a. and say, Would you like to come to your room and take some medication your doctor prescribed for you? b. accompanied by three staff members and say, Please come to your room so I can give you some medication that will help you feel more comfortable. c. and place the patient in a basket-hold and then say, I am going to take you to your room to give you an injection of medication to calm you. d. accompanied by two security guards and tell the patient, You can come to your room willingly so I can give you this medication, or the aide and I will take you there.

ANS: B A patient gains feelings of security if he or she sees that others are present to help with control. The nurse gives a simple direction, honestly states what is going to happen, and reassures the patient that the intervention will be helpful. This positive approach assumes that the patient can act responsibly and will maintain control. Physical control measures should be used only as a last resort. The security guards are likely to intimidate the patient and increase feelings of vulnerability.

.A nurse works with a person who was raped four years ago. This person says, It took a long time for me to recover from that horrible experience. Which term should the nurse use when referring to this person? a. Victim b. Survivor c. Plaintiff d. Perpetrator

ANS: B A survivor is an individual who has experience sexual assault, participated in interventions, and is moving forward in life. Victim refers to a person who experienced a recent sexual assault. Plaintiff refers to a person bringing a civil complaint to the court system. Perpetrator refers to a person who commits a crime.

A patient comes to the hospital for treatment of injuries sustained during a rape. The patient abruptly decides to decline treatment and return home. Before the patient leaves, the nurse should: a. tell the patient, You may not leave until you receive prophylactic treatment for sexually transmitted diseases. b. provide written information concerning the physical and emotional reactions that may be experienced. c. explain the need and importance of human immunodeficiency virus (HIV) testing. d. offer verbal information about legal resources.

ANS: B All information given to a patient before he or she leaves the emergency department should be in writing. Patients who are anxious are unable to concentrate and therefore cannot retain much of what is verbally imparted. Written information can be read and referred to at later times. Patients cannot be kept against their will or coerced into receiving medication as a condition of being allowed to leave. This constitutes false imprisonment

The student nurse learning about cellular regulation understands that which process occurs during the S phase of the cell cycle? a. Actual division (mitosis) b. Doubling of DNA c. Growing extra membrane d. No reproductive activity

ANS: B During the S phase, the cell must double its DNA content through DNA synthesis. Actual division, or mitosis, occurs during the M phase. Growing extra membrane occurs in the G1 phase. During the G0 phase, the cell is working but is not involved in any reproductive activity.

A new patient immediately requires seclusion on admission. The assessment is incomplete, and no prescriptions have been written. Immediately after safely secluding the patient, which action has priority? a. Provide an opportunity for the patient to go to the bathroom. b. Notify the health care provider and obtain a seclusion order. c. Notify the hospital risk manager. d. Debrief the staff.

ANS: B Emergency seclusion can be effected by a credentialed nurse but must be followed by securing a medical order within the period specified by the state and agency. The incorrect options are not immediately necessary from a legal standpoint.

A client with cancer has anorexia and mucositis, and is losing weight. The clients family members continually bring favorite foods to the client and are distressed when the client wont eat them. What action by the nurse is best? a. Explain the pathophysiologic reasons behind the client not eating. b. Help the family show other ways to demonstrate love and caring. c. Suggest foods and liquids the client might be willing to try to eat. d. Tell the family the client isnt able to eat now no matter what they bring.

ANS: B Families often become distressed when their loved ones wont eat. Providing food is a universal sign of caring, and to some people the refusal to eat signifies worsening of the condition. The best option for the nurse is to help the family find other ways to demonstrate caring and love, because with treatment-related anorexia and mucositis, the client is not likely to eat anything right now. Explaining the rationale for the problem is a good idea but does not suggest to the family anything that they can do for the client. Simply telling the family the client is not able to eat does not give them useful information and is dismissive of their concerns.

A patient has a history of impulsively acting out anger by striking others. Which would be an appropriate plan for avoiding such incidents? a. Explain that restraint and seclusion will be used if violence occurs. b. Help the patient identify incidents that trigger impulsive acting out. c. Offer one-on-one supervision to help the patient maintain control. d. Administer lorazepam (Ativan) every 4 hours to reduce the patients anxiety.

ANS: B Identifying trigger incidents allows the patient and nurse to plan interventions to reduce irritation and frustration that lead to acting out anger and to put more adaptive coping strategies eventually into practice.

When a victim of sexual assault is discharged from the emergency department, the nurse should: a. arrange support from the victims family. b. provide referral information verbally and in writing. c. advise the victim to try not to think about the assault. d. offer to stay with the victim until stability is regained.

ANS: B Immediately after the assault, rape victims are often disorganized and unable to think well or remember what they have been told. Written information acknowledges this fact and provides a solution. The incorrect options violate the patients right to privacy, evidence a rescue fantasy, and offer a platitude that is neither therapeutic nor effective.

A clinic nurse interviews an adult patient who reports fatigue, back pain, headaches, and sleep disturbances. The patient seems tense and then becomes reluctant to provide more information and hurries to leave. How can the nurse best serve the patient? a. Explore the possibility of patient social isolation. b. Have the patient complete an abuse assessment screen. c. Ask whether the patient has ever had psychiatric counseling. d. Ask the patient to disrobe; then assess for signs of physical abuse.

ANS: B In this situation, the nurse should consider the possibility that the patient is a victim of intimate partner violence. Although the patient is reluctant to discuss issues, he or she may be willing to fill out an abuse assessment screen, which would then open the door to discussion.

A client is concerned about the risk of lymphedema after a mastectomy. Which response by the nurse is best? a. You do not need to worry about lymphedema since you did not have radiation therapy. b. A risk factor for lymphedema is infection, so wear gloves when gardening outside. c. Numbness, tingling, and swelling are common sensations after a mastectomy. d. The risk for lymphedema is a real threat and can be very self-limiting.

ANS: B Infection can create lymphedema; therefore, the client needs to be cautious with activities using the affected arm, such as gardening. Radiation therapy is just one of the factors that could cause lymphedema. Other risk factors include obesity and the presence of axillary disease. The symptoms of lymphedema are heaviness, aching, fatigue, numbness, tingling, and swelling, and are not common after the surgery. Women with lymphedema live fulfilling lives.

A patient recently hospitalized for two weeks committed suicide during the night. Which initial measure will be most helpful for staff members and other patients regarding this event? a. Request the public information officer to make an announcement to the local media. b. Hold a staff meeting to express feelings and plan the care for other patients. c. Ask the patients roommate not to discuss the event with other patients. d. Quickly discharge as many patients as possible to prevent panic.

ANS: B Interventions should be aimed at helping the staff and patients come to terms with the loss and to grow because of the incident. Then, a community meeting should be scheduled to allow other patients to express their feelings and request help. Staff members should be prepared to provide additional support and reassurance to patients and should seek opportunities for peer support. The incorrect options will not control information or may result in unsafe care.

Which statement by a patient during an assessment interview should alert the nurse to the patients need for immediate, active intervention? a. I am mixed up, but I know I need help. b. I have no one for help or support. c. It is worse when you are a person of color. d. I tried to get attention before I shot myself.

ANS: B Lack of social support and social isolation increase the suicide risk. The willingness to seek help lowers the risk. Being a person of color does not suggest a higher risk; more whites commit suicide than do individuals of other racial groups. Attention seeking is not correlated with a higher risk of suicide.

A rape victim tells the emergency department nurse, I feel so dirty. Please let me take a shower before the doctor examines me. The nurse should: a. arrange for the patient to shower. b. explain that washing would destroy evidence. c. give the patient a basin of hot water and towels. d. instruct the victim to wash above the waist only.

ANS: B No matter how uncomfortable, the patient should not bathe until the forensic examination is completed. The collection of evidence is critical if the patient is to be successful in court. The incorrect options would result in the destruction of evidence or are untrue.

After receiving the hand-off report, which client should the oncology nurse see first? a. Client who is afebrile with a heart rate of 108 beats/min b. Older client on chemotherapy with mental status changes c. Client who is neutropenic and in protective isolation d. Client scheduled for radiation therapy today

ANS: B Older clients often do not exhibit classic signs of infection, and often mental status changes are the first observation. Clients on chemotherapy who become neutropenic also often do not exhibit classic signs of infection. The nurse should assess the older client first. The other clients can be seen afterward.

A client has received a dose of ondansetron (Zofran) for nausea. What action by the nurse is most important? a. Assess the client for a headache. b. Assist the client in getting out of bed. c. Instruct the client to reduce salt intake. d. Weigh the client daily before the client eats.

ANS: B Ondansetron side effects include postural hypotension, vertigo, and bradycardia, all of which increase the clients risk for injury. The nurse should assist the client when getting out of bed. Headache and fluid retention are not side effects of this drug.

Which situation constitutes consensual sex rather than rape? a. After coming home intoxicated from a party, a person forces the spouse to have sex. The spouse objects. b. A persons lover pleads to have oral sex. The person gives in but then regrets the decision. c. A person is beaten, robbed, and forcibly subjected to anal penetration by an assailant. d. A physician gives anesthesia for a procedure and has intercourse with an unconscious patient.

ANS: B Only the correct answer describes a scenario in which the sexual contact is consensual. Consensual sex is not considered rape if the participants are, at least, the age of majority.

An emergency department nurse realizes that the spouse of a patient is becoming increasingly irritable while waiting. Which intervention should the nurse use to prevent escalation of anger? a. Explain that the patients condition is not life threatening. b. Periodically provide an update and progress report on the patient. c. Explain that all patients are treated in order, based on their medical needs. d. Suggest that the spouse return home until the patients treatment is completed.

ANS: B Periodic updates reduce anxiety and defuse anger. This strategy acknowledges the spouses presence and concerns. The incorrect options are likely to increase anger because they imply that the anxiety is inappropriate.

A nurse working in the county jail interviews a man who recently committed a violent sexual assault against a woman. Which comment from this perpetrator is most likely? a. She was very beautiful .b. I gave her what she wanted. c. I have issues with my mother. d. Ive been depressed for a long time.

ANS: B Rape involves a need for control, power, degradation, and dominance over others. The correct response shows a lack of remorse or guilt, which is a common characteristic of an antisocial personality. The incorrect responses show an appreciation for women, psychological conflict, and self-disclosure, which are not expected from a perpetrator of sexual assault.

A client in the oncology clinic reports her family is frustrated at her ongoing fatigue 4 months after radiation therapy for breast cancer. What response by the nurse is most appropriate? a. Are you getting adequate rest and sleep each day? b. It is normal to be fatigued even for years afterward. c. This is not normal and Ill let the provider know. d. Try adding more vitamins B and C to your diet.

ANS: B Regardless of the cause, radiation-induced fatigue can be debilitating and may last for months or years after treatment has ended. Rest and adequate nutrition can affect fatigue, but it is most important that the client understands this is normal.

After treatment for a detached retina, a victim of intimate partner violence says, My partner only abuses me when intoxicated. Ive considered leaving, but I was brought up to believe you stay together, no matter what happens. I always get an apology, and I can tell my partner feels bad after hitting me. Which nursing diagnosis applies? a. Social isolation, related to lack of community support system b. Risk for injury, related to partners physical abuse when intoxicated c. Deficient knowledge, related to resources for escape from the abusive relationship d. Disabled family coping, related to uneven distribution of power within a relationship

ANS: B Risk for injury is the priority diagnosis because the partner has already inflicted physical injury during violent episodes. The episodes are likely to become increasingly violent. Data are not present that show social isolation or disabled family coping, although both are common among victims of violence. Deficient knowledge does not apply to this patients use of defense mechanisms.

A patient with a history of anger and impulsivity is hospitalized after an accident resulting in injuries. When in pain, the patient loudly scolds the nursing staff for not knowing enough to give me pain medicine when I need it. Which nursing intervention would best address this problem? a. Tell the patient to notify nursing staff 30 minutes before the pain returns so the medication can be prepared. b. Urge the health care provider to change the prescription for pain medication from as needed to a regular schedule. c. Tell the patient that verbal assaults on nurses will not shorten the wait for pain medication. d. Have the clinical nurse leader request a psychiatric consultation.

ANS: B Scheduling the medication at specific intervals will help the patient anticipate when the medication can be given. Receiving the medication promptly on schedule, rather than expecting nurses to anticipate the pain level, should reduce anxiety and anger. The patient cannot predict the onset of pain before it occurs.

Which rationale best explains why a nurse should be aware of personal feelings while working with a family experiencing family violence? a. Self-awareness protects ones own mental health. b. Strong negative feelings interfere with assessment and judgment. c. Strong positive feelings lead to underinvolvement with the victim. d. Positive feelings promote the development of sympathy for patients.

ANS: B Strong negative feelings cloud the nurses judgment and interfere with assessment and intervention, no matter how well the nurse tries to cover or deny personal feelings. Strong positive feelings lead to overinvolvement with the victim.

Which intervention should a nurse recommend for the distressed family and friends of someone who has committed suicide? a. Participating in reminiscence therapy b. Attending a self-help group for survivors c. Contracting for two sessions of group therapy d. Completing a psychological postmortem assessment

ANS: B Survivors need outlets for their feelings about the loss and the deceased person. Self-help groups provide peer support while survivors work through feelings of loss, anger, and guilt. Psychological postmortem assessment would not provide the support necessary to work through feelings of loss associated with the suicide of a family member. Reminiscence therapy is not geared to loss resolution. Contracting for two sessions of group therapy would probably not provide sufficient time to work through the issues associated with a death by suicide.

An adult patient assaults another patient and is restrained. One hour later, which statement by this restrained patient necessitates the nurses immediate attention? a. I hate all of you! b. My fingers are tingly. c. You wait until I tell my lawyer. d. It was not my fault. The other patient started it.

ANS: B The correct response indicates impaired circulation and necessitates the nurses immediate attention. The incorrect responses indicate that the patient has continued aggressiveness and agitation.

A survivor in the long-term reorganization phase of the rape trauma syndrome has experienced intrusive thoughts of the rape and developed a fear of being alone. Which finding demonstrates this survivor has made improvement? The survivor: a. temporarily withdraws from social situations. b. plans coping strategies for fearful situations. c. uses increased activity to reduce fear. d. expresses a desire to be with others.

ANS: B The correct response shows a willingness and ability to take personal action to reduce the disabling fear. The incorrect responses demonstrate continued ineffective coping.

An adolescent tells the school nurse, My friend threatened to take an overdose of pills. The nurse talks to the friend who verbalized the suicide threat. The most critical question for the nurse to ask would be: a. Why do you want to kill yourself? b. Do you have access to medications? c. Have you been taking drugs and alcohol? d. Did something happen with your parents?

ANS: B The nurse must assess the patients access to the means to carry out the plan and, if there is access, alert the parents to remove them from the home. The other questions may be important to ask but are not the most critical.

A victim of physical abuse by an intimate partner is treated for a broken wrist. The patient has considered leaving but says, You stay together, no matter what happens. Which outcome should be met before the patient leaves the emergency department? The patient will: a. limit contact with the abuser by obtaining a restraining order. b. name two community resources that can be contacted. c. demonstrate insight into the abusive relationship. d. facilitate counseling for the abuser.

ANS: B The only outcome indicator clearly attainable within this time is for a staff member to provide the victim with information about community resources that can be contacted. The development of insight into the abusive relationship requires time. Securing a restraining order can be quickly accomplished but not while the patient is in the emergency department. Facilitating the abusers counseling may require weeks or months.

A patient being admitted suddenly pulls a knife from a coat pocket and threatens, I will kill anyone who tries to get near me. An emergency code is called. The patient is safely disarmed and placed in seclusion. Justification for the use of seclusion is that the patient: a. evidences a thought disorder, rendering rational discussion ineffective. b. presents a clear and present danger to others. c. presents a clear escape risk. d. is psychotic.

ANS: B The patients threat to kill self or others with the knife he possesses constitutes a clear and present danger to self and others. The distractors are not sufficient reasons for seclusion.

A client tells the oncology nurse about an upcoming vacation to the beach to celebrate completing radiation treatments for cancer. What response by the nurse is most appropriate? a. Avoid getting salt water on the radiation site. b. Do not expose the radiation area to direct sunlight. c. Have a wonderful time and enjoy your vacation! d. Remember you should not drink alcohol for a year.

ANS: B The skin overlying the radiation site is extremely sensitive to sunlight after radiation therapy has been completed. The nurse should inform the client to avoid sun exposure to this area. This advice continues for 1 year after treatment has been completed. The other statements are not appropriate.

A victim of a sexual assault that occurred approximately 1 hour earlier sits in the emergency department rocking back and forth and repeatedly saying, I cant believe Ive been raped. This behavior is characteristic of which phase of the rape trauma syndrome? a. Anger phase b. Acute phase c. Outward adjustment phase d. Long-term reorganization phase

ANS: B The victims response is typical of the acute phase and evidences cognitive, affective, and behavioral disruptions. The response is immediate and does not include a display of behaviors suggestive of the outward adjustment, long-term reorganization, or anger phases.

A patient with burn injuries has had good coping skills for several weeks. Today, a new nurse is poorly organized and does not follow the patients usual schedule is. By mid-afternoon, the patient is angry and loudly complains to the nurse manager. Which is the nurse managers best response? a. Explain the reasons for the disorganization, and take over the patients care for the rest of the shift. b. Acknowledge and validate the patients distress and ask, What would you like to have happen? c. Apologize and explain that the patient will have to accept the situation for the rest of the shift. d. Ask the patient to control the anger and explain that allowances must be made for new staff members.

ANS: B When a patient with good coping skills is angry and overwhelmed, the goal is to reestablish a means of dealing with the situation. The nurse should solve the problem with the patient by acknowledging the patients feelings, validating them as understandable, apologizing if necessary, and then seeking an acceptable solution. Often patients can tell the nurse what they would like to have happen as a reasonable first step.

The student nurse caring for clients who have cancer understands that the general consequences of cancer include which client problems? (Select all that apply.) a. Clotting abnormalities from thrombocythemia b. Increased risk of infection from white blood cell deficits c. Nutritional deficits such as early satiety and cachexia d. Potential for reduced gas exchange e. Various motor and sensory deficits

ANS: B, C, D, E The general consequences of cancer include reduced immunity and blood-producing functions, altered GI structure and function, decreased respiratory function, and motor and sensory deficits. Clotting problems often occur due to thrombocytopenia (not enough platelets), not thrombocythemia (too many platelets).

A nurse is participating in primary prevention efforts directed against cancer. In which activities is this nurse most likely to engage? (Select all that apply.) a. Demonstrating breast self-examination methods to women b. Instructing people on the use of chemoprevention c. Providing vaccinations against certain cancers d. Screening teenage girls for cervical cancer e. Teaching teens the dangers of tanning booths

ANS: B, C, E Primary prevention aims to prevent the occurrence of a disease or disorder, in this case cancer. Secondary prevention includes screening and early diagnosis. Primary prevention activities include teaching people about chemoprevention, providing approved vaccinations to prevent cancer, and teaching teens the dangers of tanning beds. Breast examinations and screening for cervical cancer are secondary prevention methods.

After assessing a victim of sexual assault, which terms could the nurse use in the documentation? Select all that apply. a. Alleged b. Reported c. Penetration d. Intercourse e. Refused f. Declined

ANS: B, C, F The nurse should refrain from using pejorative language when documenting assessments of victims of sexual assault. Reported should be used instead of alleged. Penetration should be used instead of intercourse. Declined should be used instead of refused.

A rape victim tells the nurse, I should not have been out on the street alone. Which is the nurses most therapeutic response? a. Rape can happen anywhere. b. Blaming yourself only increases your anxiety and discomfort. c. You believe this would not have happened if you had not been alone? d. You are right. You should not have been alone on the street at night.

ANS: C A reflective communication technique is helpful. Looking at ones role in the event serves to explain events that the victim would otherwise find incomprehensible. The incorrect options discount the victims perceived role and interfere with further discussion.

A patient with severe injuries is irritable, angry, and belittles the nurses. As a nurse changes a dressing, the patient screams, Dont touch me! You are so stupid. You will make it worse! Which intervention uses a cognitive technique to help this patient? a. Discontinue the dressing change without comments and leave the room. b. Stop the dressing change, saying, Perhaps you would like to change your own dressing. c. Continue the dressing change, saying, Do you know this dressing change is needed so your wound will not get infected? d. Continue the dressing change, saying, Unfortunately, you have no choice. Your doctor ordered this dressing change.

ANS: C Anger is cognitively driven. The correct answer helps the patient test his cognitions and may help lower his anger. The incorrect options will escalate the patients anger by belittling or escalating the patients sense of powerlessness.

Which referral is most appropriate for a woman who is severely beaten by her husband, has no relatives or friends in the community, is afraid to return home, and has limited financial resources? a. Support group b. Law enforcement c. Womens shelter d. Vocational counseling

ANS: C Because the woman has no safe place to go, referral to a shelter is necessary. The shelter will provide other referrals as necessary.

The nursing instructor explains the difference between normal cells and benign tumor cells. What information does the instructor provide about these cells? a. Benign tumors grow through invasion of other tissue. b. Benign tumors have lost their cellular regulation from contact inhibition. c. Growing in the wrong place or time is typical of benign tumors. d. The loss of characteristics of the parent cells is called anaplasia.

ANS: C Benign tumors are basically normal cells growing in the wrong place or at the wrong time. Benign cells grow through hyperplasia, not invasion. Benign tumor cells retain contact inhibition. Anaplasia is a characteristic of cancer cells.

A confused older adult patient in a skilled care facility is sleeping. The nurse enters the room quietly and touches the bed to see if it is wet. The patient awakens and hits the nurse in the face. Which statement best explains the patients action? a. Older adult patients often demonstrate exaggerations of behaviors used earlier in life. b. Crowding in skilled care facilities increases individual tendencies toward violence. c. The patient interpreted the health care workers behavior as potentially harmful. d. This patient learned violent behavior by watching other patients act out.

ANS: C Confused patients are not always able to evaluate accurately the actions of others. This patient behaved as though provoked by the intrusive actions of the staff member.

A tearful, anxious patient at the outpatient clinic reports, I should be dead. The initial task of the nurse conducting the assessment interview is to: a. assess the lethality of a suicide plan. b. encourage expression of anger. c. establish a rapport with the patient. d. determine risk factors for suicide.

ANS: C Establishing rapport will allow the nurse to obtain relevant assessment data such as the presence of a suicide plan, the lethality of a suicide plan, and the presence of risk factors for suicide.

A college student failed two tests. Afterward, the student cried for hours and then tried to telephone a parent but got no answer. The student then gave several expensive sweaters to a roommate. Which behavior provides the strongest clue of an impending suicide attempt? a. Calling parents b. Excessive crying c. Giving away sweaters d. Staying alone in a dorm room

ANS: C Giving away prized possessions may signal that the individual thinks he or she will have no further need for the items, such as when a suicide plan has been formulated. Calling parents and crying do not provide clues to suicide, in and of themselves. Remaining in the dormitory would be an expected behavior because the student has nowhere else to go.

When assessing a patients plan for suicide, what aspect has priority? a. Patients financial and educational status b. Patients insight into suicidal motivation c. Availability of means and lethality of method d. Quality and availability of patients social support

ANS: C If a person has definite plans that include choosing a method of suicide readily available, and if the method is one that is lethal (i.e., will cause the person to die with little probability for intervention), the suicide risk is considered high. These areas provide a better indication of risk than the areas mentioned in the other options.

A patient is pacing the hall near the nurses station and swearing loudly. An appropriate initial intervention for the nurse would be to address the patient by name and say: a. Hey, whats going on? b. Please quiet down immediately. c. Id like to talk with you about how youre feeling right now. d. You must go to your room and try to get control of yourself.

ANS: C Intervention should begin with an analysis of the patient and situation. With this response, the nurse is attempting to hear the patients feelings and concerns, which leads to the next step of planning an intervention. The incorrect responses are authoritarian, creating a power struggle between the patient and nurse.

Which finding in a female client by the nurse would receive the highest priority of further diagnostics? a. Tender moveable masses throughout the breast tissue b. A 3-cm firm, defined mobile mass in the lower quadrant of the breast c. Nontender immobile mass in the upper outer quadrant of the breast d. Small, painful mass under warm reddened skin

ANS: C Malignant lesions are hard, nontender, and usually located in the upper outer quadrant of the breast and would be the priority for further diagnostic study. The other lesions are benign breast disorders. The tender moveable masses throughout the breast tissue could be a fibrocystic breast condition. A firm, defined mobile mass in the lower quadrant of the breast is a fibroadenoma, and a painful mass under warm reddened skin could be a local abscess or ductal ectasia.

With a history of breast cancer in the family, a 48-year-old female client is interested in learning about the modifiable risk factors for breast cancer. After the nurse explains this information, which statement made by the client indicates that more teaching is needed? a. I am fortunate that I breast-fed each of my three children for 12 months. b. It looks as though I need to start working out at the gym more often. c. I am glad that we can still have wine with every evening meal. d. When I have menopausal symptoms, I must avoid hormone replacement therapy.

ANS: C Modifiable risk factors can help prevent breast cancer. The client should lessen alcohol intake and not have wine 7 days a week. Breast-feeding, regular exercise, and avoiding hormone replacement are also strategies for breast cancer prevention.

Which medication should a nurse administer to provide immediate intervention for a psychotic patient whose aggressive behavior continues to escalate despite verbal intervention? a. lithium (Eskalith) b. trazodone (Desyrel) c. olanzapine (Zyprexa) d. valproic acid (Depakene)

ANS: C Olanzapine is a short-acting antipsychotic drug that is useful in calming angry, aggressive patients regardless of their diagnosis. The other drugs listed require long-term use to reduce anger. Lithium is for patients with bipolar disorder. Trazodone is for patients with depression, insomnia, or chronic pain. Valproic acid is for patients with bipolar disorder or borderline personality disorder.

A patient diagnosed with pneumonia has been hospitalized for 4 days. Family members describe the patient as a difficult person who finds fault with others. The patient verbally abuses nurses for providing poor care. The most likely explanation for this behavior lies in: a. poor childrearing that did not teach respect for others. b. automatic thinking, leading to cognitive distortion. c. personality style that externalizes problems. d. delusions that others wish to deliver harm.

ANS: C Patients whose personality style causes them to externalize blame see the source of their discomfort and anxiety as being outside themselves. They displace anger and are often unable to soothe themselves. The incorrect options are less likely to have a bearing on this behavior.

A client is admitted with superior vena cava syndrome. What action by the nurse is most appropriate? a. Administer a dose of allopurinol (Aloprim). b. Assess the clients serum potassium level. c. Gently inquire about advance directives. d. Prepare the client for emergency surgery.

ANS: C Superior vena cava syndrome is often a late-stage manifestation. After the client is stabilized and comfortable, the nurse should initiate a conversation about advance directives. Allopurinol is used for tumor lysis syndrome. Potassium levels are important in tumor lysis syndrome, in which cell destruction leads to large quantities of potassium being released into the bloodstream. Surgery is rarely done for superior vena cava syndrome.

A 35-year-old woman is diagnosed with stage III breast cancer. She seems to be extremely anxious. What action by the nurse is best? a. Encourage the client to search the Internet for information tonight. b. Ask the client if sexuality has been a problem with her partner. c. Explore the idea of a referral to a breast cancer support group. d. Assess whether there has been any mental illness in her past.

ANS: C Support for the diagnosis would be best with a referral to a breast cancer support group. The Internet may be a good source of information, but the day of diagnosis would be too soon. The nurse could assess the frequency and satisfaction of sexual relations but should not assume that there is a problem in that area. Assessment of mental illness is not an appropriate action.

Four clients are receiving tyrosine kinase inhibitors (TKIs). Which of these four clients should the nurse assess first? a. Client with dry, itchy, peeling skin b. Client with a serum calcium of 9.2 mg/dL c. Client with a serum potassium of 2.8 mEq/L d. Client with a weight gain of 0.5 pound (1.1 kg) in 1 day

ANS: C TKIs can cause electrolyte imbalances. This potassium level is very low, so the nurse should assess this client first. Dry, itchy, peeling skin can be a problem in clients receiving biologic response modifiers, and the nurse should assess that client next because of the potential for discomfort and infection. This calcium level is normal. TKIs can also cause weight gain, but the client with the low potassium level is more critical.

A client is starting hormonal therapy with tamoxifen (Nolvadex) to lower the risk for breast cancer. What information needs to be explained by the nurse regarding the action of this drug? a. It blocks the release of luteinizing hormone. b. It interferes with cancer cell division. c. It selectively blocks estrogen in the breast. d. It inhibits DNA synthesis in rapidly dividing cells.

ANS: C Tamoxifen (Nolvadex) reduces the estrogen available to breast tumors to stop or prevent growth. This drug does not block the release of luteinizing hormone to prevent the ovaries from producing estrogen; leuprolide (Lupron) does this. Chemotherapy agents such as ixabepilone (Ixempra) interfere with cancer cell division, and doxorubicin (Adriamycin) inhibits DNA synthesis in susceptible cells.

A nurse uses the SAD PERSONS scale to interview a patient. This tool provides data relevant to: a. current stress level. b. mood disturbance. c. suicide potential. d. level of anxiety.

ANS: C The SAD PERSONS tool evaluates 10 major risk factors in suicide potential: sex, age, depression, previous attempt, ethanol use, rational thinking loss, social supports lacking, organized plan, no spouse, and sickness. The tool does not have appropriate categories to provide information on the other options listed.

Which scenario predicts the highest risk for directing violent behavior toward others? a. Major depressive disorder with delusions of worthlessness b. Obsessive-compulsive disorder; performing many rituals c. Paranoid delusions of being followed by a military attack team d. Completion of alcohol withdrawal and beginning a rehabilitation program

ANS: C The correct answer illustrates the greatest disruption of ability to perceive reality accurately. People who feel persecuted may strike out against those believed to be persecutors. The patients identified in the distractors have better reality-testing ability.

A nurse and patient construct a no-suicide contract. Select the preferable wording for the contract. a. I will not try to harm myself during the next 24 hours. b. I will not make a suicide attempt while I am hospitalized. c. For the next 24 hours, I will not kill or harm myself in any way. d. I will not kill myself until I call my primary nurse or a member of the staff.

ANS: C The correct answer leaves no loopholes. The wording about not harming oneself and not making an attempt leaves loopholes or can be ignored by the patient who thinks, I am not going to harm myself, I am going to kill myself, or I am not going to attempt suicide, I am going to commit suicide. A patient may call a therapist and leave the telephone to carry out the suicidal plan.

After an assault by a patient, a nurse has difficulty sleeping, startles easily, and is preoccupied with the incident. The nurse says, I dread facing potentially violent patients. Which response would be the most urgent reason for this nurse to seek supervision? a. Startle reactions b. Difficulty sleeping c. A wish for revenge d. Preoccupation with the incident

ANS: C The desire for revenge signals an urgent need for professional supervision to work through anger and counter the aggressive feelings. The distractors are normal in a person who has been assaulted. Nurses are usually relieved with crisis intervention and follow-up designed to give support, help the individual regain a sense of control, and make sense of the event.

When working with rape victims, immediate care focuses first on: a. collecting evidence. b. notifying law enforcement. c. helping the victim feel safe. d. documenting the victims comments.

ANS: C The first focus of care is helping the victim feel safe. An already vulnerable individual may view assessment questions and the physical procedures as intrusive violations of privacy and even physically threatening. The patient might decline to have evidence collected or to involve law enforcement.

An adult tells the nurse, My partner abuses me most often when drinking. The drinking has increased lately, but I always get an apology afterward and a box of candy. Ive considered leaving but havent been able to bring myself to actually do it. Which phase in the cycle of violence prevents the patient from leaving? a. Tension building b. Acute battering c. Honeymoon d. Recovery

ANS: C The honeymoon stage is characterized by kindly, loving behaviors toward the abused spouse when the perpetrator feels remorseful. The victim believes the promises and drops plans to leave or seek legal help. The tension-building stage is characterized by minor violence in the form of abusive verbalization or pushing. The acute battering stage involves the abuser beating the victim. The violence cycle does not include a recovery stage.

The nurse is examining a womans breast and notes multiple small mobile lumps. Which question would be the most appropriate for the nurse to ask? a. When was your last mammogram at the clinic? b. How many cans of caffeinated soda do you drink in a day? c. Do the small lumps seem to change with your menstrual period? d. Do you have a first-degree relative who has breast cancer?

ANS: C The most appropriate question would be one that relates to benign lesions that usually change in response to hormonal changes within a menstrual cycle. Reduction of caffeine in the diet has been shown to give relief in fibrocystic breast conditions, but research has not found that it has a significant impact. Questions related to the clients last mammogram or breast cancer history are not related to the nurses assessment.

A nurse has taught a client about dietary changes that can reduce the chances of developing cancer. What statement by the client indicates the nurse needs to provide additional teaching? a. Foods high in vitamin A and vitamin C are important. b. Ill have to cut down on the amount of bacon I eat. c. Im so glad I dont have to give up my juicy steaks. d. Vegetables, fruit, and high-fiber grains are important.

ANS: C To decrease the risk of developing cancer, one should cut down on the consumption of red meats and animal fat. The other statements are correct.

A cognitively impaired patient has been a widow for 30 years. This patient is frantically trying to leave the unit, saying, I have to go home to cook dinner before my husband arrives from work. To intervene with validation therapy, the nurse should first say: a. You must come away from the door. b. You have been a widow for many years. c. You want to go home to prepare your husbands dinner? d. Was your husband angry if you did not have dinner ready on time?

ANS: C Validation therapy meets the patient where she or he is at the moment and acknowledges the patients wishes. Validation does not seek to redirect, reorient, or probe. The incorrect options do not validate the patients feelings.

A nurse assesses a patient who reports a 3-week history of depression and crying spells. The patient says, My business is bankrupt, and I was served with divorce papers. Which subsequent statement by the patient alerts the nurse to a concealed suicidal message? a. I wish I were dead. b. Life is not worth living. c. I have a plan that will fix everything. d. My family will be better off without me.

ANS: C Verbal clues to suicide may be overt or covert. The incorrect options are overt references to suicide. The correct option is more veiled. It alludes to the patients suicide as being a way to fix everything but does not say it outright.

A college student failed two examinations. The student cried for hours and then tried to call a parent but got no answer. The student then suspended access to his social networking web site. Which suicide risk factors are present? Select all that apply. a. History of earlier suicide attempt b. Co-occurring medical illness c. Recent stressful life event d. Self-imposed isolation e. Shame or humiliation

ANS: C, D, E Failing examinations in the academic major constitutes a recent stressful life event. Shame and humiliation related to the failure can be hypothesized. The inability to contact parents can be seen as a recent lack of social support, as can the roommates absence from the dormitory. Terminating access to ones social networking site represents self-imposed isolation. This scenario does not provide data regarding a history of an earlier suicide attempt, a family history of suicide, or of co-occurring medical illness.

A nurse assesses the health status of soldiers returning from Afghanistan. Screening for which health problems will be a priority? Select all that apply. a. Schizophrenia b. Eating disorder c. Traumatic brain injury d. Oppositional defiant disorder e. Post-traumatic stress disorder

ANS: C, E Traumatic brain injury and post-traumatic stress disorder each occur in approximately 20% of soldiers returning from Afghanistan. Some soldiers have both problems. The incidence of disorders identified in the distractors would be expected to parallel the general population.

A nurse is assessing a female client who is taking progestins. What assessment finding requires the nurse to notify the provider immediately? a. Irregular menses b. Edema in the lower extremities c. Ongoing breast tenderness d. Red, warm, swollen calf

ANS: D All clients receiving progestin therapy are at risk for thromboembolism. A red, warm, swollen calf is a manifestation of deep vein thrombosis and should be reported to the provider. Irregular menses, edema in the lower extremities, and breast tenderness are common side effects of the therapy.

A nurse works with clients who have alopecia from chemotherapy. What action by the nurse takes priority? a. Helping clients adjust to their appearance b. Reassuring clients that this change is temporary c. Referring clients to a reputable wig shop d. Teaching measures to prevent scalp injury

ANS: D All of the actions are appropriate for clients with alopecia. However, the priority is client safety, so the nurse should first teach ways to prevent scalp injury.

A rape victim asks an emergency department nurse, Maybe I did something to cause this attack. Was it my fault? Which response by the nurse is the most therapeutic? a. Pose questions about the rape, helping the patient explore why it happened. b. Reassure the victim that the outcome of the situation will be positive. c. Make decisions for the victim because of the temporary confusion. d. Support the victim to separate issues of vulnerability from blame.

ANS: D Although the victim may have made choices that increased vulnerability, the victim is not to blame for the rape. The incorrect options either suggest the use of a nontherapeutic communication technique or do not permit the victim to restore control. No confusion is evident.

A person was abducted and raped at gunpoint. The nurse observes this person is confused, talks rapidly in disconnected phrases, and is unable to concentrate or make simple decisions. What is the persons level of anxiety? a. Weak b. Mild c. Moderate d. Severe

ANS: D Anxiety is the result of a personal threat to the victims safety and security. In this case, the persons symptoms of rapid, dissociated speech, confusion, and indecisiveness indicate severe anxiety. Weak is not a level of anxiety. Mild and moderate levels of anxiety allow the person to function at a higher level.

The nurse has taught a client with cancer ways to prevent infection. What statement by the client indicates that more teaching is needed? a. I should take my temperature daily and when I dont feel well. b. I will wash my toothbrush in the dishwasher once a week. c. I wont let anyone share any of my personal items or dishes. d. Its alright for me to keep my pets and change the litter box.

ANS: D Clients should wash their hands after touching their pets and should not empty or scoop the cat litter box. The other statements are appropriate for self-management.

A nurse interviews a person abducted and raped at gunpoint by an unknown assailant. The person says, I cant talk about it. Nothing happened. I have to forget! What is the persons present coping strategy? a. Somatic reaction b. Repression c. Projection d. Denial

ANS: D Disbelief is a common finding during the acute stage following sexual assault. Denial is evidence of the disbelief. This mechanism may be unconsciously used to protect the person from the emotionally overwhelming reality of rape. The patients statements do not reflect somatic symptoms, repression, or projection.

An employee has recently been absent from work on several occasions. Each time, this employee returns to work wearing dark glasses. Facial and body bruises are apparent. During the occupational health nurses interview, the employee says, My partner beat me, but it was because there are problems at work. What should the nurses next action be? a. Call the police. b. Arrange for hospitalization. c. Call the adult protective agency. d. Document injuries with a body map.

ANS: D Documentation of the injuries provides a basis for possible legal intervention. The abused adult will need to make the decision to involve the police. Because the worker is not an older adult and is competent, the adult protective agency is unable to assist. Admission to the hospital is not necessary.

A client with cancer is admitted to a short-term rehabilitation facility. The nurse prepares to administer the clients oral chemotherapy medications. What action by the nurse is most appropriate? a. Crush the medications if the client cannot swallow them. b. Give one medication at a time with a full glass of water. c. No special precautions are needed for these medications. d. Wear personal protective equipment when handling the medications.

ANS: D During the administration of oral chemotherapy agents, nurses must take the same precautions that are used when administering IV chemotherapy. This includes using personal protective equipment. These medications cannot be crushed, split, or chewed. Giving one at a time is not needed.

Which individual in the emergency department should be considered at the highest risk for completing suicide? a. An adolescent Asian-American girl with superior athletic and academic skills who has asthma b. A 38-year-old single African-American female church member with fibrocystic breast disease c. A 60-year-old married Hispanic man with 12 grandchildren who has type 2 diabetes d. A 79-year-old single white man with cancer of the prostate gland

ANS: D High-risk factors include being an older adult, single, and male and having a co-occurring medical illness. Cancer is one of the somatic conditions associated with increased suicide risk. Protective factors for African-American women and Hispanic individuals include strong religious and family ties. Asian Americans have a suicide rate that increases with age.

Which understanding about individuals who attempt suicide will help a nurse plan the care for a suicidal patient? Every suicidal person should be considered: a. mentally ill. b. intent on dying. c. cognitively impaired. d. experiencing hopelessness.

ANS: D Hopelessness is the characteristic common among people who attempt suicide. The incorrect options reflect myths about suicide. Not all who attempt suicide are intent on dying. Not all are mentally ill or cognitively impaired.

What is the primary motivator for most rapists? a. Anxiety b. Need for humiliation c. Overwhelming sexual desires d. Desire to humiliate or control others

ANS: D Rape is not a crime of sex; rather, it is a crime of power, control, and humiliation. The perpetrator wishes to subjugate the victim. The dynamics listed in the other options are not the major motivating factors for rape.

When a patients aggression quickly escalates, which principle applies to the selection of nursing interventions? a. Staff members should match the patients affective level and tone of voice. b. Ask the patient what intervention would be most helpful. c. Immediately use physical containment measures. d. Begin with the least restrictive measure possible.

ANS: D Standards of care require that staff members use the least restrictive measure possible. This becomes the guiding principle for intervention. Physical containment is seldom the least restrictive measure. Asking the out-of-control patient what to do is rarely helpful. It may be an effective strategy during the preassaultive phase but is less effective during escalation.

A client is in the oncology clinic for a first visit since being diagnosed with cancer. The nurse reads in the clients chart that the cancer classification is TISN0M0. What does the nurse conclude about this clients cancer? a. The primary site of the cancer cannot be determined. b. Regional lymph nodes could not be assessed. c. There are multiple lymph nodes involved already. d. There are no distant metastases noted in the report.

ANS: D TIS stands for carcinoma in situ; N0 stands for no regional lymph node metastasis; and M0 stands for no distant metastasis.

A 37-year-old Nigerian woman is at high risk for breast cancer and is considering a prophylactic mastectomy and oophorectomy. What action by the nurse is most appropriate? a. Discourage this surgery since the woman is still of childbearing age. b. Reassure the client that reconstructive surgery is as easy as breast augmentation. c. Inform the client that this surgery removes all mammary tissue and cancer risk. d. Include support people, such as the male partner, in the decision making.

ANS: D The cultural aspects of decision making need to be considered. In the Nigerian culture, the man often makes the decisions for care of the female. Women with a high risk for breast cancer can consider prophylactic surgery. If reconstructive surgery is considered, the procedure is more complex and will have more complications compared to a breast augmentation. There is a small risk that breast cancer can still develop in the remaining mammary tissue.

A patient is hospitalized after an arrest for breaking windows in the home of a former domestic partner. The history reveals childhood abuse by a punitive parent, torturing family pets and an arrest for disorderly conduct. Which nursing diagnosis has priority? a. Risk for injury b. Post-trauma response c. Disturbed thought processes d. Risk for other-directed violence

ANS: D The defining characteristics for Risk for other-directed violence include a history of being abused as a child, having committed other violent acts, and demonstrating poor impulse control. The defining characteristics for the other diagnoses are not present in this scenario.

A patient sits in silence for 20 minutes after a therapy appointment, appearing tense and vigilant. The patient abruptly stands and paces back and forth, clenching and unclenching fists, and then stops and stares in the face of a staff member. The patient is: a. demonstrating withdrawal. b. working through angry feelings. c. attempting to use relaxation strategies. d. exhibiting clues to potential aggression.

ANS: D The description of the patients behavior shows the classic signs of someone whose potential for aggression is increasing.

A nurse counsels a patient with recent suicidal ideation. Which is the nurses most therapeutic comment? a. Lets make a list of all your problems and think of solutions for each one. b. Im happy youre taking control of your problems and trying to find solutions. c. When you have bad feelings, try to focus on positive experiences from your life. d. Lets consider which problems are most important and which are less important.

ANS: D The nurse helps the patient develop effective coping skills. He or she assists the patient to reduce the overwhelming effects of problems by prioritizing them. The incorrect options continue to present overwhelming approaches to problem solving.

A nurse answers a suicide crisis line. A caller says, I live alone in a home several miles from my nearest neighbors. I have been considering suicide for 2 months. I have had several drinks and now my gun is loaded. Im going to shoot myself in the heart. How would the nurse assess the lethality of this plan? a. No risk b. Low level c. Moderate level d. High level

ANS: D The patient has a highly detailed plan, a highly lethal method, the means to carry it out, lowered impulse control because of alcohol ingestion, and a low potential for rescue.

A nurse working a rape telephone hotline should focus communication with callers to: a. arrange long-term counseling. b. serve as a sympathetic listener. c. obtain information to relay to the local police. d. explain immediate steps that a victim of rape should take.

ANS: D The telephone counselor establishes where the victim is and what has happened and provides the necessary information to enable the victim to decide what steps to take immediately. Long-term aftercare is not the focus until immediate problems are resolved. The victim remains anonymous. The incorrect options are inappropriate or incorrect because counselors should be empathic rather than sympathetic.

A new nurse has been assigned a client who is in the hospital to receive iodine-131 treatment. Which action by the nurse is best? a. Ensure the client is placed in protective isolation. b. Hand off a pregnant client to another nurse. c. No special action is necessary to care for this client. d. Read the policy on handling radioactive excreta.

ANS: D This type of radioisotope is excreted in body fluids and excreta (urine and feces) and should not be handled directly. The nurse should read the facilitys policy for handling and disposing of this type of waste. The other actions are not warranted.

A severely depressed patient who has been on suicide precautions tells the nurse, I am feeling a lot better, so you can stop watching me. I have taken too much of your time already. Which is the nurses best response? a. I wonder what this sudden change is all about. Please tell me more. b. I am glad you are feeling better. The team will consider your request. c. You should not try to direct your care. Leave that to the treatment team. d. Because we are concerned about your safety, we will continue with our plan.

ANS: D When a patient seeks to have precautions lifted by professing to feel better, the patient may be seeking greater freedom in which to attempt suicide. Changing the treatment plan requires careful evaluation of outcome indicators by the staff. The incorrect options will not cause the patient to admit to a suicidal plan, do not convey concern for the patient, or suggest that the patient is not a partner in the care process.

Select the most helpful response for a nurse to make when a patient being treated as an outpatient states, I am considering suicide. a. Im glad you shared this. Please do not worry. We will handle it together. b. I think you should admit yourself to the hospital to get help. c. We need to talk about the good things you have to live for. d. Bringing this up is a very positive action on your part.

ANS: DT his response gives the patient reinforcement and validation for making a positive response rather than acting out the suicidal impulse. It gives neither advice nor false reassurance, and it does not imply stereotypes such as, You have a lot to live for. It uses the patients ambivalence and sets the stage for more realistic problem-solving strategies.


Ensembles d'études connexes

AP US Government Amendments, Court Cases, Random Stuff (Unit 3)

View Set

History Ch. 15-19 Quiz Questions

View Set

Agency and who a licensee can represent!

View Set

Hospitality and Tourism Cluster Exam

View Set

Nursing 2 Chapter 12 Book questions

View Set

Lesson 13 - countercurrent, concurrent, crosscurrent

View Set